You are helping your friend move a new refrigerator into his kitchen. You apply a horizontal force of 275 N in the positive x direction to try and move the 61 kg refrigerator. The coefficient of static friction is 0.58. (a) How much static frictional force does the floor exert on the refrigerator

Answers

Answer 1

Answer:

f = 347.08 N

Explanation:

The frictional force exerted by the floor on the refrigerator is given as follows:

[tex]f = \mu R = \mu W[/tex]

where,

f = frictional force = ?

μ = coefficient of static friction = 0.58

W = Weight of refrigerator = mg

m = mass of refrigerator = 61 kg

g = acceleration due to gravity = 9.81 m/s²

Therefore,

[tex]f = \mu mg\\f = (0.58)(61\ kg)(9.81\ m/s^2)\\[/tex]

f = 347.08 N


Related Questions

0. The temperature of source is 500K with source energy 2003, what is the temperature of sink with sink energy 100 J? a. 500 K b. 300 K c. 250 K d. 125 K​

Answers

Answer:

c. 250k

Explanation:

The temperature of the sink is approximately 250 K.

To find the temperature of the sink, we can use the formula for the efficiency of a heat engine:

Efficiency = 1 - (Temperature of Sink / Temperature of Source)

Given that the temperature of the source (T_source) is 500 K and the source energy (Q_source) is 2003 J, and the sink energy (Q_sink) is 100 J, we can rearrange the formula to solve for the temperature of the sink (T_sink):

Efficiency = (Q_source - Q_sink) / Q_source

Efficiency = (2003 J - 100 J) / 2003 J

Efficiency = 1903 J / 2003 J

Efficiency = 0.9497

Now, plug the efficiency back into the first equation to solve for T_sink:

0.9497 = 1 - (T_sink / 500 K)

T_sink / 500 K = 1 - 0.9497

T_sink / 500 K = 0.0503

Now, isolate T_sink:

T_sink = 0.0503 * 500 K

T_sink = 25.15 K

Since the temperature should be in Kelvin, we round down to the nearest whole number, which is 25 K. Thus, the temperature of the sink is approximately 250 K.

To learn more about sink energy, here

https://brainly.com/question/10483137

#SPJ2

1. On each of your equipotential maps, draw some electric field lines with arrow heads indicating the direction of the field. (Hint: At what angle do field lines intersect equipotential lines?) Draw sufficient field lines that you can "see" the electric field.

Answers

Answer:

The angle between the electric field lines and the equipotential surface is 90 degree.

Explanation:

The equipotential surfaces are the surface on which the electric potential is same. The work done in moving a charge from one point to another on an equipotential surface is always zero.

The electric field lines are always perpendicular to the equipotential surface.

As

[tex]dV = \overrightarrow{E} . d\overrightarrow{r}\\\\[/tex]

For equipotential surface, dV = 0 so

[tex]0 = \overrightarrow{E} . d\overrightarrow{r}\\\\[/tex]

The dot product of two non zero vectors is zero, if they are perpendicular to each other.

An object is moving from north to south what is the direction of the force of friction of the object

Answers

Answer:

North

Explanation:

Friction is a reaction force against the direction of movement. So since the direction of movement is south the friction would be opposite and move north.

Answer:

South To North

Explanation:

Frictional force acts in the direction opposite to the direction of motion of a body. Because the object is moving from north to south, the direction of frictional force is from south to north

* A ball is projected horizontally from the top of
a building 19.6m high.
a, How long when the ball take to hit the ground?
b, If the line joining the point of projection to
the point where it hits the ground is 45
with the horizontal. What must be the
initial velocity of the ball?
c,with what vertical verocity does the ball strike
the grounds? (9= 9.8 M152)​

Answers

Explanation:

Given

Ball is projected horizontally from a building of height [tex]h=19.6\ m[/tex]

time taken to reach ground is given by

[tex]\text{Cosidering vertical motion}\\\Rightarrow h=ut+0.5at^2\\\Rightarrow 19.6=0+0.5\times 9.8t^2\\\Rightarrow t^2=4\\\Rightarrow t=2\ s[/tex]

(b) Line joining the point of projection and the point where it hits the ground makes an angle of [tex]45^{\circ}[/tex]

From the figure, it can be written

[tex]\Rightarrow \tan 45^{\circ}=\dfrac{h}{x}\\\\\Rightarrow x=h\cdot 1\\\Rightarrow x=19.6[/tex]

Considering horizontal motion

[tex]\Rightarrow x=u_xt\\\Rightarrow 19.6=u_x\times 4\\\Rightarrow u_x=4.9\ m/s[/tex]

(c) The vertical velocity with which it strikes the ground is given by

[tex]\Rightarrow v^2-u_y^2=2as\\\Rightarrow v^2-0=2\times 9.8\times 19.6\\\Rightarrow v=\sqrt{384.16}\\\Rightarrow v=19.6\ m/s[/tex]

Thus, the ball strikes with a vertical velocity of [tex]19.6\ m/s[/tex]

Explanation:

Given

Ball is projected horizontally from a building of height  

time taken to reach ground is given by

(b) Line joining the point of projection and the point where it hits the ground makes an angle of  

From the figure, it can be written

Considering horizontal motion

(c) The vertical velocity with which it strikes the ground is given by

Thus, the ball strikes with a vertical velocity of

The cart travels the track again and now experiences a constant tangential acceleration from point A to point C. The speeds of the cart are 10.6 ft/s at point A and 15.6 ft/s at point C. The cart takes 4.00 s to go from point A to point C, and the cart takes 1.80 s to go from point B to point C. What is the cart's speed at point B

Answers

Answer:

a) [tex]a_{avg}=1.25ft/s^2[/tex]

b) [tex]v_b=13.35ft/s[/tex]

Explanation:

From the question we are told that:

Speed at point A [tex]v_A=10.6ft/s[/tex]

Speed at point C [tex]v_C=15.6ft/s[/tex]

Time from Point A to C [tex]T_{ac}=4.00s[/tex]

Time from Point B to C [tex]T_{bc}=1.80s[/tex]

Generally the equation for acceleration From A to B is mathematically given by

 [tex]a_{avg}=\frac{v_c-v_a}{\triangle t}[/tex]

 [tex]a_{avg}=\frac{15.6-10.6}{4.0 }[/tex]

 [tex]a_{avg}=1.25ft/s^2[/tex]

Generally the equation for cart speed at B is mathematically given by

 [tex]a_{avg}=\frac{v_c-v_a}{T_{bc}}[/tex]

 [tex]v_b=v_c-a_{avg}*T_{bc}[/tex]

 [tex]v_b=15.6ft/s-(1.25ft/s^2)(1.80)[/tex]

  [tex]v_b=13.35ft/s[/tex]

What are the differences among elements, compounds, and mixtures?

Answers

Answer:

Elements have a characteristic number of electrons and protons.Both Hydrogen(H) and oxygen(O) are two different elements.

••••••••••••••••

Compounds are chemical substances where the atoms atoms of two different elements are combined together. It is made of .Hydrogen(H) and Oxygen(O) both qr4 naturally gases,but they react to form water(H2O),which is liquid compound.

•••••••••••••••

A mixture is made of atleast two parts》 solid,liquid or gas.The difference is that it's not a chemical substance that's bonded by other elements. 

------------------------------

Hope it helps...

Have a great day!!!

Answer: Elements have a characteristic number of electrons and protons. Both Hydrogen(H) and oxygen(O) are two different elements. Compounds are chemical substances where the atoms atoms of two different elements are combined together. It is made of.Hydrogen(H) and Oxygen(O) both qr4 naturally gases, but they react to form water(H2O), which is liquid compound. A mixture is made of at least two parts solid, liquid, or gas. The difference is that it's not a chemical substance that's bonded by other elements.

While an object near the earths surface is in free fall, its
A) velocity increases
B) acceleration increases

Answers

Answer:

a

Explanation:

The rate of change of an object's location with relation to a reference point is its velocity, which is dependent on time. when an object is dropped from space at rest (t = 0) under the influence of gravity, the velocity of the object changes and increases with time while the acceleration decreases.

One hazard of space travel is the debris left by previous missions. There are several thousand objects orbiting Earth that are large enough to be detected by radar, but there are far greater numbers of very small objects, such as flakes of paint. Calculate the force exerted by a 0.100-mg chip of paint that strikes a spacecraft window at a relative speed of 4.00×10^3 m/s, given the collision lasts 6.00×10^8s.

Answers

Answer:

F = 6666.7 N

Explanation:

Given that,

Mass of a chip, m = 0.1 mg

Initial speed, u = 0

Final speed,[tex]v=4\times 10^{3}\ m/s[/tex]

Time of collision,[tex]t=6\times 10^{-8}\ s[/tex]

We know that,

Force, F = ma

Put all the values,

[tex]F=\dfrac{m(v-u)}{t}\\\\F=\dfrac{0.1\times 10^{-6}\times (4\times 10^3-0)}{6\times 10^{-8}}\\\\F=6666.7\ N[/tex]

So, the required force is 6666.7 N.

Q 26.12: Assume current flows in a cylindrical conductor in such a way that the current density increases linearly with radius, from zero at the center to 1.0 A/m2 at the surface of the conductor. If the conductor has a cross sectional area of 1.0 m2, what can you say about the current in this conductor

Answers

Answer:

The current is 0.67 A.

Explanation:

Density, J = 1 A/m^2

Area, A = 1 m^2

Let the radius is r. And outer is R.

Use the formula of current density

[tex]I = \int J dA = \int J 2\pi r dr\\\\I = \int_{0}^{R}\frac{2\pi r^2}{R} dr\\\\I = \frac{2 \pi R^2}{3}.... (1)Now A = \pi R^2\\\\1 =\pi R^2\\\\R^2 = \frac{1}{\pi}\\\\So, \\\\I = \frac{2\pi}{3}\times \frac{1}{\pi}\\\\I = 0.67 A[/tex]

if 145kl of energy is added to water, what mass of water can be heated from 35C to 100C then vaporized at 100C

Answers

Answer:

m = 0.057 kg = 57 g

Explanation:

Energy Added to Water = Heat added to raise the temperature of water + Heat used to vaporize water

[tex]E = mC\Delta T + mH\\E = m(C\Delta T + H)[/tex]

where,

E = Energy added to water = 145 KJ

m = mass of water = ?

C = specific heat capacity of water = 4.2 KJ/kg.°C

ΔT = change in temperature = 100°C - 35°C = 65°C

H = Latent heat of vaporization of water = 2260 KJ/kg

Therefore,

[tex]145\ KJ = m[(4.2\ KJ/kg.^oC)(65^oC)+2260\ KJ/kg]\\\\145\ KJ = m(2533\ KJ/kg)\\\\m = \frac{145\ KJ}{2533\ KJ/kg}[/tex]

m = 0.057 kg = 57 g

The mass of water that can be heated is equal to 0.527 kilograms.

Given the following data:

Quantity of energy = 145 kJ = 145,000 Joules.Initial temperature = 35.0°CFinal temperature = 100.0°C

Scientific data:

Specific heat capacity of water = 4200 J/kg°CLatent heat of vaporization of water = 2260 KJ/kg

To calculate the mass of water that can be heated:

The quantity of energy and heat.

Note: The quantity of energy added to water is equal to the quantity of heat used to vaporize water and the quantity of heat that is added to raise the temperature of water.

Mathematically, this is given by this expression:

[tex]E=mc\theta + mH\\\\E= m(c\theta + H)[/tex]

Making m the subject of formula, we have:

[tex]m=\frac{E}{c\theta + H}[/tex]

Substituting the parameters into the formula, we have;

[tex]m=\frac{145000}{[42000\times (100-35)] + 2260}\\\\m=\frac{145000}{(4200\times 65) + 2260}\\\\m=\frac{145000}{273000 + 2260}\\\\m=\frac{145000}{275260}[/tex]

Mass, m = 0.527 kilograms.

Read more on quantity of energy here: https://brainly.com/question/13439286

the magnitude of the magnetic field at point p for a certain electromagnetic wave is 2.21. What is the magnitude of the elctic field for that wave at P

Answers

Answer:

[tex]6.63\times 10^8\ N/C[/tex]

Explanation:

Given that,

The magnitude of magnetic field, B = 2.21

We need to find the magnitude of the electric field. Let it is E. So,

[tex]\dfrac{E}{B}=c\\\\E=Bc[/tex]

Put all the values,

[tex]E=2.21\times 3\times 10^8\\\\=6.63\times 10^8\ N/C[/tex]

So, the magnitude of the electric field is equal to [tex]6.63\times 10^8\ N/C[/tex].

A cylindrical tank with radius 7 m is being filled with water at a rate of 2 m3/min. How fast is the height of the water increasing (in m/min)?

Answers

Answer:

0.013 m/min

Explanation:

Applying,

dV/dt = (dh/dt)(dV/dh)............. Equation 1

Where

V = πr²h................ Equation 2

Where V = volume of the tank, r = radius, h = height.

dV/dh = πr²............ Equation 3

Substitute equation 3 into equation 1

dV/dt = πr²(dh/dt)

From the question,

Given: dV/dt = 2 m³/min, r = 7 m, π = 3.14

Substitute these values into equation 3

2 = (3.14)(7²)(dh/dt)

dh/dt = 2/(3.14×7²)

dh/dt = 0.013 m/min

A pump lifts 400 kg of water per hour a height of 4.5 m .
Part A
What is the minimum necessary power output rating of the water pump in watts?
Express your answer using two significant figures.
Part B
What is the minimum necessary power output rating of the water pump in horsepower?
Express your answer using two significant figures.

Answers

Answer:

Power = Work / Time

P = 400 kg * 9.8 m/s * 4.5 m / 3600 sec = 4.9 J/s = 4.9 Watts

Also, 4.9 Watts / (746 Watts / Horsepower) = .0066 Hp

A box-shaped metal can has dimensions 8 in. by 4 in. by 10 in. high. All of the air inside the can is removed with a vacuum pump. Assuming normal atmospheric pressure outside the can, find the total force on one of the 8-by-10-in. sides

Answers

Answer:

The force on the side is 5252 N.

Explanation:

Area, A =  8 in x 10 in = 80 in^2 = 0.052 m^2

height, h = 10 in

The force on the area is

F = P x A

where, P is the atmospheric pressure and A is the area.  

P = 1.01 x 10^5 Pa

Force = 1.01 x10^5 x 0.052 = 5252 N

The free-fall acceleration at the surface of planet 1 is 22 m/s^2. The radius and the mass of planet 2 are twice those of planet 1. What is the free-fall acceleration on planet 2?

Answers

Answer:

g₂ = 11 m/s²

Explanation:

The value of free-fall acceleration on the surface of a planet is given by the following formula:

[tex]g = \frac{Gm}{r^2}[/tex]

where,

g = free-fall acceleration

G = Universal Gravitational Constant

m = mass of the planet

r = radius of planet

FOR PLANET 1:

[tex]g_1 = \frac{Gm_1}{r_1^2}\\\\\frac{Gm_1}{r_1^2} = 22 m/s^2[/tex] --------------------- equation (1)

FOR PLANET 2:

[tex]g_2 = \frac{Gm_2}{r_2^2}\\\\g_2 = \frac{G(2m_1)}{(2r_1)^2}\\\\g_2 = \frac{1}{2}\frac{Gm_1}{r_1^2}\\\\[/tex]

using equation (1):

[tex]g_2 = \frac{g_1}{2}\\\\g_2 = \frac{22\ m/s^2}{2}[/tex]

g₂ = 11 m/s²

lamp in a child's Halloween costume flashes based on an RC discharge of a capacitor through its resistance. The effective duration of the flash is 0.360 s, during which it produces an average 0.690 W from an average 3.00 V. (a) What energy does it dissipate

Answers

Energy = (power) x (time)

Energy = (0.69 W) x (0.36 sec)

Energy = 0.25 Joule

Betelgeuse (in Orion) has a parallax of 0.00451 + 0.00080 arcsec,as measured by the Hipparcos satellite. What is the distance to Betelgeuse, and what is the uncertainty in that measurement?

Answers

Betelgeuse, the bright red star in the constellation of Orion the Hunter, is in the end stage of its stellar life. Astronomers have long thought it will someday explode to become a supernova. In late 2019 and early 2020, Betelgeuse generated a lot of

We have that  the distance to Betelgeuse, and the uncertainty in that measurement is

[tex]d=(221.7\pm39.33)pc[/tex]Uncertainty U = 0.00080

From the Question we are told that

Betelgeuse (in Orion)  has a parallax of 0.00451 + 0.00080

Generally

[tex]Distance\ in\ parsecs =\frac{ 1}{(parallax\ measured\ in\ arcseconds}[/tex]

Where

Parallax [tex]P =0.00451[/tex]

Uncertainty [tex]U = 0.00080[/tex]

Generally the equation for the distance  is mathematically given as

[tex]d=(\frac{1}{P}pc\pm(\frac{U}{P}*100\%))[/tex]

Therefore

[tex]d=(\frac{1}{0.00451}pc\pm(\frac{0.00080}{0.00451}*100\%))[/tex]

[tex]d=(221.7\pm39.33)pc[/tex]

For more information on this visit

https://brainly.com/question/12319416?referrer=searchResults

Which one is the better material to use for an inexpensive compass? hard iron, soft iron, or any conductor ​

Answers

Answer:

Soft iron

Explanation:

A disk rotates about its central axis starting from rest and accelerates with constant angular acceleration. At one time it is rotating at 10 rev/s; 60 revolutions later, its angular speed is 15 rev/s. Calculate
(a) the angular acceleration,
(b) the time required to complete the 60 revolutions,
(c) the time required to reach the 10 rev/s angular speed, and
(d) the number of revolutions from rest until the time the disk reaches the 10 rev/s angular speed.

Answers

Explanation:

Given:

[tex]\omega_0[/tex] = 10 rev/s = [tex]20\pi\:\text{rad/s}[/tex]

[tex]\omega[/tex] = 15 rev/s = [tex]30\pi\:\text{rad/s}[/tex]

[tex]\theta[/tex] = 60 rev = [tex]120\pi\:\text{rads}[/tex]

a) the angular acceleration [tex]\alpha[/tex] is given by

[tex]\alpha = \dfrac{\omega^2 - \omega_0^2}{2\theta}[/tex]

[tex]\:\:\:\:\:\:\:=\dfrac{(30\pi)^2 - (20\pi)^2}{240\pi} = 6.5\:\text{rad/s}^2[/tex]

b) [tex]t = \dfrac{\omega - \omega_0}{\alpha} = \dfrac{30\pi - 20\pi}{6.5} = 4.8\:\text{s}[/tex]

c) [tex]t = \dfrac{\omega - \omega_0}{\alpha}[/tex]

[tex]=\dfrac{20\pi - 0}{6.5} = 9.7\:\text{s}[/tex]

d)[tex]\theta = \frac{1}{2}\alpha t^2[/tex]

[tex]\:\:\:\:\:\:\:=\frac{1}{2}(6.5\:\text{rad/s}^2)(9.7\:\text{s})^2 = 305.8\:\text{rad}[/tex]

[tex]\:\:\:\:\:\:\:= 48.7\:\text{revs}[/tex]

if Petrol diesel etc catches fire one should never try to extinguish in using water why?​

Answers

Answer:

because both petrol and diesel are oil

Explanation:

oil floats on water that's why if we will try to extinguish with water so the fire will float on water

hope u like my answer

please mark methe brainest

Find the force on a negative charge that is placed midway between two equal positive charges. All charges have the same magnitude.

Answers

Answer: The force on a negative charge that is placed midway between two equal positive charges is zero when all charges have the same magnitude.

Explanation:

Let us assume that

[tex]q_{1} = q_{2} = +q[/tex]

[tex]q_{3} = -q[/tex]

As [tex]q_{3}[/tex] is the negative charge and placed midway between two equal positive charges ([tex]q_{1}[/tex] and [tex]q_{2}[/tex]).

Total distance between [tex]q_{1}[/tex] and [tex]q_{2}[/tex] is 2r. This means that the distance between [tex]q_{1}[/tex] and [tex]q_{3}[/tex], [tex]q_{2}[/tex] and [tex]q_{3}[/tex] = d = r

Now, force action on charge [tex]q_{3}[/tex] due to [tex]q_{1}[/tex] is as follows.

[tex]F_{31} = k(\frac{q_{1} \times q_{3}}{d^{2}})[/tex]

where,

k = electrostatic constant = [tex]9 \times 10^{9} Nm^{2}/C^{2}[/tex]

Substitute the values into above formula as follows.

[tex]F_{31} = k(\frac{q_{1} \times q_{3}}{d^{2}})\\= 9 \times 10^{9} (\frac{q \times (-q)}{r^{2}})\\= - 9 \times 10^{9} (\frac{q^{2}}{r^{2}})[/tex] ... (1)

Similarly, force acting on [tex]q_{3}[/tex] due to [tex]q_{1}[/tex] is as follows.

[tex]F_{32} = k \frac{q_{2}q_{3}}{d^{2}}\\= -9 \times 10^{9} \frac{q^{2}}{r^{2}}\\[/tex]   ... (2)

As both the forces represented in equation (1) and (2) are same and equal in magnitude. This means that the net force acting on charge [tex]q_{3}[/tex] is zero.

Thus, we can conclude that the force on a negative charge that is placed midway between two equal positive charges is zero when all charges have the same magnitude.

The Lamborghini Huracan has an initial acceleration of 0.85g. Its mass, with a driver, is 1510 kg. If an 80 kg passenger rode along, what would the car's acceleration be?​

Answers

Answer:

7.9 [tex]\frac{m}{s^{2} }[/tex]

Explanation:

Take the fact that mass is inversely proportional to accelertation:

m ∝ a

Therefore m = a, but because we are finding the change in acceleration, we would set our problem up to look more like this:

[tex]\frac{m_{1} }{m_{2} } = \frac{a_{2} }{a_{1} } \\[/tex]

Using algebra, we can rearrange our equation to find the final acceleration, [tex]a_{2}[/tex]:

[tex]a_{2} = \frac{a_{1}*m_{1} }{m_{2} } \\[/tex]

Before plugging everything in, since you are being asked to find acceleration, you will want to convert 0.85g to m/s^2. To do this, multiply by g, which is equal to 9.8 m/s^2:

0.85g * 9.8 [tex]\frac{m }{s^{2} }[/tex] = 8.33 [tex]\frac{m }{s^{2} }[/tex]

Plug everything in:

7.9 [tex]\frac{m }{s^{2} }[/tex] = [tex]\frac{ 8.33\frac{m}{s^{2} }*1510kg }{1590kg}[/tex]

(1590kg the initial weight plus the weight of the added passenger)

I need help with this physics question.

Answers

Answer:

5.04 m

Explanation:

You are told that the homeowner wants to increase their fences by 34 percent meaning Original+ 34 percent. If the original is 100 percent, then the new fence size will be 134 % of the original. You are given the original which is 3.76 meters, to find new fence size 1.34 * 3.76m to get 5.0384 meters, rounded to 5.04 m.

Answer:

5.0384m

Explanation:

% increase = 100 x (Final - Initial / | initial | )

( |~~| Bars indicate absolute value since you can't have a negative height)

Every object around you is attracted to you. In fact, every object in the galaxy is attracted to every other object in the galaxy.

a. True
b. False

Answers

Answer:

True

Explanation:

With the gravitational pull that our planets have, we are able to remain in orbit. This demonstrates how every object in the galaxy is attracted to every other object. Every object in the universe that has mass exerts a gravitational pull on every other mass. We as humans do it too, but since our force isn't strong, we don't have much of an effect. I hope this helped and please don't hesitate to reach out with more questions!

TIME REMAINING
45:43
What are possible units for impulse? Check all that apply.

kg • m
kg • meters per second
N • s
N • m
StartFraction Newtons times meters per second EndFraction

Answers

Answer:

n.m maby

Explanation:

i think or its kg m/s

Answer:

answer (B) & (C)

Explanation:

kg • /N • s

A magnetohydrodynamic (MHD) drive works by applying a magnetic field to a fluid which is carrying an electric current.

a. True
b. False

Answers

Answer:

True

Explanation:

A magnetohydrodynamic drive or MHD accelerator is a method which is used for propelling the vehicles using only by applying the electric and magnetic fields. It has no moving parts. It accelerates an electrically conductive propellant (liquid or gas) with magnetohydrodynamics.

Its working principle is same as an electric motor except that in an MHD drive, the moving rotor is replaced by the fluid acting directly as the propellant.

An MHD accelerator is reversible.

So,  the statement is true.

A horizontal, mass spring system undergoes simple harmonic motion. which of the following statements is correct reguarding the mass in the system when it is located at its maximum distance from the equilibrium position?
a. The acceleration of the mass is zero.
b. The potential energy of the spring attached to the mass is at a minimum.
c. The total mechanical energy of the mass is zero.
d. The kinetic energy of the mass is at a maximum.
e. The speed of the mass is zero.

Answers

Answer:

Option (e)

Explanation:

A body executing SHM moves to and fro or back and forth  about its mean position.

When the particle is at mean position, its velocity is maximum and when it is at extreme position its velocity is zero.

So, when it is at maximum distance:

a.

The acceleration is maximum.

b.

The potential energy is maximum.

c.

The total mechanical energy is non zero.

d.

The kinetic energy is zero.

e. The speed is zero. Correct

Pure water is an example of alan
A. Insulator
B. Metalloid
C. Conductor
D. Nonmetal

Answers

A. Insulator is the right answer

Answer: I think its A or C I'm not sure though sorry.

It takes 130 J of work to compress a certain spring 0.10m. (a) What is the force constant of this spring? (b) To compress the spring an additional 0.10 m, does it take 130 J, more than 130 J or less than 130 J? Verify your answer with a calculation.

Answers

Explanation:

Given that,

Work done to stretch the spring, W = 130 J

Distance, x = 0.1 m

(a) We know that work done in stretching the spring is as follows :

[tex]W=\dfrac{1}{2}kx^2\\\\k=\dfrac{2W}{x^2}\\\\k=\dfrac{2\times 130}{(0.1)^2}\\\\k=26000\ N/m[/tex]

(b) If additional distance is 0.1 m i.e. x = 0.1 + 0.1 = 0.2 m

So,

[tex]W=\dfrac{1}{2}kx^2\\\\W=\dfrac{1}{2}\times 26000\times 0.2^2\\\\W=520\ J[/tex]

So, the new work is more than 130 J.

High-speed stroboscopic photographs show that the head of a -g golf club is traveling at m/s just before it strikes a -g golf ball at rest on a tee. After the collision, the club head travels (in the same direction) at m/s. Find the speed of the golf ball just after impact.

Answers

The question is incomplete. The complete question is :

High-speed stroboscopic photographs show that the head of a 200 g golf club is traveling at 60 m/s just before it strikes a  50 g golf ball at rest on a tee. After the collision, the club head travels (in the same direction) at 40 m/s. Find the speed of the golf ball just after impact.

Solution :

We know that momentum = mass x velocity

The momentum of the golf club before impact = 0.200 x 60

                                                                             = 12 kg m/s

The momentum of the ball before impact is zero. So the total momentum before he impact is 12 kg m/s. Therefore, due to the conservation of momentum of the two bodies after the impact is 12 kg m/s.

Now the momentum of the club after the impact is = 0.2 x 40

                                                                                    = 8 kg m/s

Therefore the momentum of the ball is = 12 - 8

                                                                = 4 kg m/s

We know momentum of the ball, p = mass x velocity

                                                     4 = 0.050 x velocity

∴ Velocity =  [tex]$\frac{4}{0.050}$[/tex]

                 = 80 m/s

Hence the speed of the golf ball after the impact is 80 m/s.

Other Questions
7^9 = ????? My daughter needs help with this really quick! A baseball league is holding registration for both a men's league and a women's league. Only a total of 400 players can register, and each team consists of exactly 10 players. If 24 women's teams have already registered, which inequality could be used to find m, the number of men's teams that can register? A. 24(10) + 10m < 400 B. 24(10) + 10m > 400 C. 24(10 + 10m) > 400 D. 24(10 + 10m) < 400 Three forces of magnitude 10N, 5N and 4N act on an object in the directions North, West and East respectively. Find the magnitude and directions of their resultant Please help I dont understand Can someone help me with this math homework please! find x. need help w these 2 questions in the pic, thanks! simplify the following.(2x-y)(x+2y) please answer all of this questions as fast as possible Assume that consumers' incomes and the number of sellers in the market for good A (a normal good) both decrease. Based upon this information we can conclude, with certainty, that equilibrium: What are line breaks in poetryWhere the poet chooses to end the linesWhere the poet separates the sectionsPauses the natural flow of the linesChoices about words that end sentences. Read the excerpt from Rogue Heart by Axie Oh. Then choose the correct way to complete the sentence.The denotative meaning of the word constant is "to be steady," and the connotative meaning of constant in the excerpt isA. lastingB. HopelessC. PointessD. Boring which of the following best states the central idea of the gettysburg address The three types of ocean floor sediments are terrigenous, biogenous, and _____. A sample of the salaries of assistant professors on the business faculty at a local university revealed a mean income of $100,000 with a standard deviation of $10,000. Assume that salaries follow a bell-shaped distribution. Use the empirical rule: a. Approximately what percentage of the salaries fall between $90,000 and $110,000?b. Approximately what percentage of the salaries fall between $80,000 and $120,000? c. Approximately what percentage of the salaries are greater than $120,000? See attached writing an equation for the graphy= Which of the following statements is not true? A. Avoiding contact with the blood of another person can prevent communicable diseases from spreading, but other bodily fluids pose no risk. B. Maintaining doctor visits for regular examinations can help you to detect communicable diseases early and avoid spreading them to others. C. Safe food handling and preparation of food is one way that you can help to prevent contracting and spreading some communicable diseases. D. To prevent communicable diseases from spreading, you should wash your hands after touching animals and make sure pets receive routine veterinary care. the foreign policy is the? On the leeside of mountains, bands of heavy snow in some areas with trace amounts of snow in between is likely the result of: solve: 7m + 12 = 4m + 78 find missing side of triangle